Final 6/12/2010

Pongo los resultados:

1) a) El gradiente de la compuesta es
\nabla h(1,2) = (4,13)
La recta normal es
r(t) = (1,2,1) + t(4,13,-1)
e intersecta al plano pedido en
r(\frac{-2}{13}) = (\frac{5}{13}, 0, \frac{15}{13})

b) La divergencia es div(f) = 2z
El flujo saliente es \frac{184}{3} \pi

2) b) y = \frac{x^3}{3} + \frac{5}{3}

3) La circulación es \frac{-8}{3}
Lo resuelvo:
Calcule la circulación de f a lo largo de la curva C desde (-2,2,z_0) hasta (0,1,z_1) cuando C es la intersección de las superficies de ecuaciones: z+x=y, z-x=2y^2-y; siendo f(x,y,z) = (y,2x,z)

Sumando las dos ecuaciones
2z = 2y^2
Restando las dos ecuaciones
2x = 2y - 2y^2

Luego parametrizo la curva como
g(t) = (t - t^2, t, t^2) con 1 \leq t \leq 2
g(1) = (0, 1, 1) = B
g(2) = (-2, 2, 4) = A
(está orientada al revés, pero no importa le cambio el signo a la circulación y listo)

Luego la circulación pedida es
- \int_1^2 f(g(t)) g'(t) dt
- \int_1^2 (t, 2t-2t^2, t^2) (1-2t, 1, 2t) dt
- \int_1^2 t - 2t^2 + 2t - 2t^2 + 2t^3 dt
- \int_1^2 3t - 4t^2 + 2t^3 dt = - \frac{8}{3}
según wolfram.

4) El volúmen es 16\pi

Esta entrada fue publicada en Ejercicios de Final con respuesta. Guarda el enlace permanente.

53 respuestas a Final 6/12/2010

  1. Franco dijo:

    El ej. 3) lo hice varias veces y me da -8/3.

    Probablemente sea error mío, pero si alguno lo resuelve y le da el mismo resultado, avise.

    Saludos.

  2. Franco dijo:

    Y otra cosita, me agarró una duda con respecto al ej. 2) b)

    Cuando se realizan las últimas integrales, siempre se toma una constante «del lado de la x», pero si se tomara la constante del otro lado, quedaría el número opuesto. En este caso, sería -5/3 en vez de 5/3.

    ¿Hay que tener en cuenta eso?

    Saludos.

  3. Fabián Pereyra dijo:

    Una preguntita, en el 1)a) cómo calculaste el -1??

  4. Fabián Pereyra dijo:

    Hola Franco, el ej. 3) lo hice de dos manera diferentes, o sea, con dos parametrizaciones distintas y llegué al mismo resultado que vos, es decir -\dysplaystyle\frac{8}{3}

    PD: Casi todos me dieron resultados diferentes, ahora los paso en latex y vemos donde está el error.

    Saludos.

  5. Franco dijo:

    Buenísimo entonces, Fabián.

    El 2) y el 4) me dan igual, pero el 3) me da [tex]54\pi[/tex]

    Sobre el ej. 1) a) que preguntabas, yo tenía la misma duda y me respondieron esto en un foro de matemática, fijate si te sirve:

    http://rinconmatematico.com/foros/index.php?topic=41222.0

  6. Andrea dijo:

    Hola Damian, el pto 3 de este final me da -8/3. La curva parametrizada me queda (t-t^2,t,t^2) y los limites de integracion entre 2 y 1. Si podes por favor, me decis si hay algun error en como parametrize o los limites de integracion?
    Gracias!!

    Saludos
    Andrea

    • dami dijo:

      Holas,
      La parametrización la veo bien, lo que no me gusta es que decís que va de 2 a 1 (tenés que tener cuidado con el signo ya que debería ir de menor a mayor)
      Este mes estoy muy complicado y no tengo la resolución de este ejercicio a mano, si me copiás la resolución de tu integral (vos o Franco o cualquiera de los que les dió -8/3) me fijo si lo veo bien.
      Saludos,
      Damián.

    • ezerwiman dijo:

      Coincido con Andrea y los demas, yo lo hice entre 2 y 1 como dice ella es decir

      \int^1_5 f(g(t)) g'(t)dt\,dt
      con lo siguiente
      g(t)=(t(1-t),t,t^2) \; g'(t)=(1-2t,1,2t)
      f(g(t))=(t,2t(1-t),t^2)
      entonces queda la integral asi
      \oint_\C f dg=\int^1_5 (t,2t(1-t),t^2) (1-2t,1,2t)\,dt
      \oint_\C f dg=\int^1_5 (3t-4t^2+2t^3)\,dt = \dfrac {-8}{3}
      \oint_\C f dg=\int^1_5 (3t-4t^2+2t^3)\,dt = \dfrac {-8}{3}
      bue no encuentro como poner barrow en LAtex pero queda algo asi
      \oint_\C f dg=\dfrac{3}{2}t^2-\dfrac{4}{3}t^3+\dfrac{t^4}{2}
      $latex \oint_\C f dg= \dfrac {2}{3}-\dfrac{10}{3}\dfrac{-8}{3}

    • ezerwiman dijo:

      bue no salio todo peor es entre 2 y 1 no se porque me quedo 5 en el latex

  7. Franco dijo:

    Fijate los comentarios Andrea, varios lo hicimos y da -8/3. Saludos.

  8. Andrea dijo:

    Hola!!!, el pto b no me da…Queria saber si hay algun error en esto que estoy haciendo: derivo xy=cx + 1 y me queda
    y + xy’=c luego reemplazo y’ por -1/y’ (para hallar la ec. dif. de la flia ortogonal), integro y con el dato que pasa por (1,2) obtengo c…
    ¿Como aplico el dato «f es conservativo»?, se que si es conservativo entonces tiene funcion potencial pero no me doy cuenta como aplicarlo desconociendo f
    Gracias!!
    Saludos,
    Andrea

    • lucho dijo:

      andrea yo el b lo hice asi:

      la expresion xy=cx+1 la despejo a :

      y – 1/x = c –> esta seria la expresion de las curvas de nivel de la funcion potencial, osea las lineas equipotenciales.
      U(x,y) = k

      el campo vectorial entonces es = (U’x , U’y)

      que queda: (1/x^2 , 1) …. ahora… para sacar la expresion de las lineas de campo hay que igular

      y’ = Q / P (No estoy seguro de donde sale esto, la verdad, lo tengo en la carpeta) , donde Q es el segundo miembro del campo, y P el primero:

      y’ = 1 / (1 / x^2 ) >> y’ = x^2 >> y = (x^3 / 3) + k

      entonces ahora reemplazas por 1,2 para sacar k:

      2 = 1/3 + k >>> k = 5/3…. finalmente

      y = (x^3 / 3) + 5/3

      en cuanto al 3… a mi tambien me da -8/3… la parametrizacion me queda:
      x=-t^2+t x= -2t+1
      C: y=t C’: y=1
      z=t^2 z=2t

      con 2<t<1

      saludos.

  9. Alejandro dijo:

    me dio -8/3 el 3) !!

  10. Andrea dijo:

    Gracias lucho! esto de y’ = Q / P no lo tenia…

    Saludos!

    • ezerwiman dijo:

      Gente coincido con la resolucion de Lucho y agrego sacado del Flax Vol 3 pag 436

      Si f en con campo vectorial se llaman lineas del campo f a las curvas incluidas en R^n cuya recta tangente en cada punto tiene la direccion de f en ese punto donde si

      g(t)=(x(t);y(t))
      es la ecuacion de las lineas de campo, para cada punto de g(t) la direccion de la recta tangente dada por
      g'(t)=(x'(t);y'(t))
      debe coincidir con la direccion del vector
      f(x,y)=(P(x,y),Q(x,y)) en dicho punto
      entonces ambos vectores, en el mismo punto tienen direcciones coincidentes, por lo tanto sus respectivas componenetes deben ser proporcionales.

      \dfrac{x'(t)}{P(x,y)}=\dfrac{y'(t)}{Q(x,y)}
      luego como
      $dg(t)=g'(t)dt$
      queda
      \dfrac{x'(t)dt}{P(x,y)}=\dfrac{y'(t)dt}{Q(x,y)}
      que es
      \dfrac{dx}{P(x,y)}=\dfrac{dy}{Q(x,y)}
      quedando
      \dfrac{dy}{dx}=\dfrac{Q(x,y)}{P(x,y)}
      cuya resolucion nos permite obtener las lineas de campo

    • ezerwiman dijo:

      otra cosa que me di cuenta es que igual sin saber todo eso sale retomo desde donde empieza Lucho

      y–\dfrac{1}{x}= c
      y=dfrac {1}{x}
      y'=dfrac{-1}{x^2}
      donde reemplazo Y' por la ortogonal Yo'=dfrac{-1}{Y'}
      aca estoy hallando las lineas de campo porque unas con otras son ortogonales entonces Yo'=x^2
      Yo'=dfrac{x^3}{3}+c
      reemplazo en (1,2)
      2=\dfrac {1} {3} +c
      c=\dfrac 5 3

      y el resultado es el mismo
      Yo que seria la funcion osea pongamos Y=\dfrac{x^3}{3}+\dfrac {5}{3}

      saludos

  11. stefi dijo:

    alguien me puede decir como hizo el 4? porque me da 64pi y no 16pi como dicen

  12. ezerwiman dijo:

    Gente, yo realice el 1b y me da muy diferente… lo hice con coordenadas esfericas, alguno de los que le dio igual que a Dami me puede decir como lo hizo? les dejo lo que yo hice.

    La interseccion me da en los Z negativos porque de la curva saco

    6-(x^2+y^2)=0
    x^2+y^2=6

    los limites de integracion son en esfericas
    0<2\tita<2\pi
    \dfrac1{3}{4}\pi<w< \dfrac{\pi}{2}  latez 0<R<\sqrt{6}

    Haciedno asiluego reemplazo z por
    z=Rcosw e integrando me queda   latex \sigma=9\pi$
    si alguno lo hizo igual o sabe como lo hizo Dami por favor tirenme una punta asi lo resuelvo yo de esa manera,

    gracias

  13. ezerwiman dijo:

    uh bue salio re mal a ver:
    limites eran

    para w 3/4 pi y pi /2
    para R entre 0 y raiz(6)
    y tita entre 0 y 2pi

    y me da
    9\pi

  14. Rubén dijo:

    El ejercicio 4 me daba -16 pi, usando coordenadas cartesianas, y estaba todo aparentemente bien. Buscando distintas tablas de integrales en internet descubrí que la integral nº 247( x^2(Raíz(a^2 – x^2)) ) de la tabla de integrales de la UTN no está bien, les dejó acá la correcta ( nº 52).

    Haz clic para acceder a integrales.pdf

  15. Guido dijo:

    Damian: la definicion del ejercicio teorico 2)a) lo vi en varios finales, pero no encuentro ningun desarrollo teorico para dicho tema. ¿Esto se refiere pura y exclusivamente a la definicion, a lo que significa S.G y S.P?¿O existe alguna hipotesis o desarrollo teorico?

  16. Mariel dijo:

    Adhiero a Guido!

  17. Guido dijo:

    Mariel: ya lo encontre esto. Es la definicion de S.G y S.P. Por ej:
    Solucion General: la sn gral. de una E.D es una fcion. que verifica a dicha ec y que posee constantes esenciales y arbitrarias.
    Saludos!

    • Mariel dijo:

      Si yo con palabras respondia , pero pense que era algo un poco mas especifico! pero si es eso nada mas mejor!, Gracias!!

    • dami dijo:

      Mariel y Guido,
      Las definiciones de SG, SP, y SS, están en esta parte de la wikipedia en español.
      Aunque está un poco mejor y mas resumido en esta parte de la wikipedia en inglés.

      Resumiendo:
      Solución General: es una familia de funciones diferenciables que verifican la EDO, y tiene tantas constantes arbitrarias como el orden de la EDO.
      Solución Particular: función diferenciable que verifica la EDO y se puede obtener dando valores a las constantes de la SG.
      Solución Singular: es una función diferenciable que verifica la EDO pero no puede obtenerse asignando valores a las constantes de la SG. (No todas las EDO presentan SS)

  18. En el 1.b lo hice con cilindricas? Los limites me quedaron, tita entre 0 y 2pi, p entre 0 y sqrt(6) y z entre p y 6-p^2, estan bien? O me conviene meter esfericas. Las superficies son un casquete esferico y un cono, es verdad? Gracias.

  19. En el 1.b lo hice con cilindricas. Los limites me quedaron, tita entre 0 y 2pi, p entre 0 y sqrt(6) y z entre p y 6-p^2, estan bien? O me conviene meter esfericas. Las superficies son un casquete esferico y un cono, es verdad? Gracias.

  20. En el ejercicio 4, cuales son los limites de integracion? Yo lo hice en cilindricas pero me da otra cosa… gracias!

    • sergio dijo:

      Hola Juan Manuel, a ver para el 4 si tomamos

      g(r,t,y)=(r\cos t,y, r\sen t )

      tenemos que

      0<r<4 \quad 0<y<r^2\cos^2 t \quad 0<t<\dfrac{pi}{2}

      cualquier duda ;), y me gusta la idea del grupo de estudio te mando mensaje al mail que me pasaste 😉

      saludos

    • sergio dijo:

      la fórmula que no se ve corresponde a las coordenadas cilindricas g(r,t,y) , lastima que no se pueda previsualizar el mensaje antes de enviarlo

      saludos

  21. Sergio , fijate que en el 1.b los limites de p van de 0 a 2 como puse yo y no de 0 a 3 como mencionás vos, era un error de calculo el mio, en el desarrollo de la integral pero mis limites estaban OK, fijate sino despues lo vemos cuando nos juntemos, abrazo, JM!

  22. sergio dijo:

    Hola JM , tuve una errata en la transcripcion 0<r<2

    saludos

  23. Alex dijo:

    en el 1b me dio http://www.wolframalpha.com/input/?i=int+r%28%286-r^2%29^2-%28r^2%29%29+drdt+with+r+from+0+to+3+%2C+t+from+0+to+2pi con r perteneciente [0, 3], t (0 a 2pi) si se ve la proyeccion al plano xy de la intercepcion de las superfices es una circuferencia de radio 3.
    con z son las superfices de abajo con el cono y arriba con el paraboloide, hice cambio de variable y le agrege el jacobiano r…pero me dio algo diferente 81/2 pi

    • dami dijo:

      Alex,
      Copio la integral que aparece en el wolframalpha: \int_0^{2\pi} \int_0^3 r ((6 - r^2)^2 - r^2) dr dt = \frac{81 \pi}{2}
      Hay varios errores. Primero no se porqué haces una integral doble (tenés que integrar sobre el cuerpo, así que tenés que hacer una integral triple. De todas formas parece que queres integrar el cuerpo haciendo «techo menos piso», eso funcionaría para un volúmen, pero en este caso el integrando es 2z (que te olvidás), además en ese caso tenés mal el techo (va con raíz cuadrada y no al cuadrado), y además si sacás la intersección de las superficies ves que el radio va hasta 2 y no hasta 3.

  24. Alex dijo:

    Disculpa si era una circuferencia de radio 2 http://www.wolframalpha.com/input/?i=int+r%28%286-r^2%29^2-%28r^2%29%29+drdt+with+r+from+0+to+2+%2C+t+from+0+to+2pi ahora si me da, claro hice techo piso tuve en cuenta 2Z, y luego hice cambio de variable por eso la integral doble, solo que la intecepcion de las superficies la calcule mal..gracias.

  25. fernando dijo:

    Hola disculpen, podrian explicar como dedujeron que el p esta entre 0 y 2? como sacaron la interseccion? gracias!

  26. Como dijeron arriba, el 3 a mi tambien me da -8/3

  27. Gisele dijo:

    Hola, en el 1.a puede ser que la x del punto intersección sea -5/3??

  28. javier dijo:

    Damian, el punto 2 a) , aparte de la teoria de SG y SP, tiene una desmostracion. Me dirias una punta para encargarlo. Gracias

    • Gisele dijo:

      Hola, yo lo que hice es que como son SP, satisfacen la EDO, entonces reemplacé y1 e y2 en la ecuación a comprobar.
      Luego sumé ambas ecuaciones y las agrupé quedando
      a(y»1+y»2) +b(y’1+y’2)+c(y1+y2)=2f(x)

      Como y=y1+y2
      derivando y’=y’1+y’2
      y derivando otra vez y»=y»1+y»2

      Luego reemplazás en a(y»1+y»2) +b(y’1+y’2)+c(y1+y2)=2f(x)
      y queda ay» +by’+cy=2f(x)

      Por otro lado como y es SP, satisface la EDO y queda ay» +by’+cy=2f(x)
      que es a lo que querías llegar.

      Si tenés el FLAX, fijate que en vol 3 hay algo muy parecido.
      Espero te sirva!

Deja un comentario